¿De qué número 64 es el 80%?
I have 5 eggs, I broke 2, I cooked 2 and I ate 2, how many do I have left?
Answer:
Step-by-step explanation:
To decipher how many eggs you have left, we must read the statement well.
I have 5 eggs if I break 2 eggs it is to cook them and when they are cooked they will be eaten, so we simply do the following:
5 -2= 3
Now you only have 3 eggs left.
Step-by-step explanation:
Given:
5 eggs
Required:
Number of eggs after breaking, cooking and eating 2.
Solution:
I won't count the eggs as 6.
Broken=cooked=eaten=2
5-2=3
Hope it helps ;) ❤❤❤
Choose two expressions that equal 12−(−33) .
A
12+(−33)
B
12+33
C
−12+(−33)
D
33−(−12)
E
12−33
F
−33−(−12)
\(\textsf{Hey \: there!}\)
\(\bf{Choose \: TWO \: expressions \: that \: are \: EQUAL} \\ \bf{to \: 12 - ( - 33)} \\ \\ \\ \bullet \: \bf{12 - ( - 33) = 45}\)
\(\mathsf{It \: cant \: a \: because \: it \: gives \: you \: - 21}\)
\(\textsf{It \: can \: be \: b \: because \: it \: gives \: us \: 45}\)
\(\textsf{It \: cant \: be \: c \: because \: it \: gives \: you \: - 45}\)
\(\textsf{It \: can \: be \: d \: because \: it \: gives \: you \: 45}\)
\(\mathsf{It \: cant \: be \: e \: because \: it \: gives \: you \: -21}\)
\(\mathsf{It \: cant \: be \: f \: because \: it \: gives \: us \: - 21}\)
\(\boxed{\mathsf{To \: sum \: it \: all \: up \: your \: answers \: are: \: \boxed{ \bf {b.12 + 33 \: and \: \: d. \: \: 33 - ( - 12)}}}}\)
Good luck on your assignment and enjoy your day!~
\(\frak{LoveYourselfFirst:)}\)
Fill in the missing term on each line. Simplify any fractions.
4t + 11 = 19
4t =
Subtract 11 from both sides
t =
Divide both sides by 4
Answer:
Step-by-step explanation:
4t + 11 = 19
Subtract 11 from both sides
4t = 19 - 11
4t = 8
Divide both sides by 4
t = 8/4
t = 2
given the experimental data, what is the activation energy, ea, (in kj/mol) for this reaction? hint: r
The activation energy for the given reaction is166 KJ/mol. (Option B)
Activation energy refers to the minimum amount of extra energy that is required by a reacting molecule to be converted into product. It is the minimum amount of energy required to energize or activate atoms or molecules so that they can undergo a chemical reaction or transformation. In order to calculate the activation energy (Ea), the Arrhenius equation is used which is given by:
k=Ae^((-E)/RT)
where k is the rate of chemical reaction, A is a constant depending on the chemicals involved, E is the activation energy, R is the universal gas constant, and T is the temperature.
The equation can be expressed as
ln〖k=ln〖A-(E/RT)lne〗 〗
2.303 log〖k=2.303 log〖A-E/RT〗 〗
log〖k=log〖A-E/2.303RT〗 〗
As there are two temperatures and two rate constants given, the Arrhenius equation is rearranged:
log〖k2/k1=E/2.303(1/T1〗-1/T2)
According to the data,
k1 = 2.7 x 10^-4
T1 = 600 K
k2 = 3.5 x 10^-3
T2 = 650 K
R = 8.314
log〖(3.5 x10^(-3))/(2.7 x 10^(-4) )=E/(2.303*8.314)(1/600〗-1/650)
E = 166208 J/mol or 166 KJ
Note: The question is incomplete. The complete question probably is: Calculate the activation energy (Ea) in kJ/mol for a reaction if the rate constant (k) is 2.7 × 10^-4 (M^−1sec^−1) at 600 K and 3.5 × 10^−3 (M−^1sec^−1) at 650 K. a. −166 kJ/mol b. 166 kJ/mol c. 1.64 kJ/mol d. 1.66 × 10^5 kJ/mol. (R
Learn more about Activation energy:
https://brainly.com/question/26724488
#SPJ4
9. plsss help, will give brianlist
Answer:
\(1. \: \: \: \: \frac{1}{5} \\ 2. \: \: \: \: - \frac{3}{2} \\ 3. \: \: \: \: \frac{1}{2} \)
Step-by-step explanation:
1. In the first diagram we simply count the number of squares along the rise and the number of squares along the run as slope equals rise/run
2. we use the slope formula
\( \frac{y2 - y1}{x2 - x1} = \frac{ - 2 - 1}{ 1- ( - 1)} = \frac{ - 3}{2} = - \frac{3}{2} \)
3. pick any two points and apply the slope formula again
\(( - 1. \: \: 1) \: and \: (3. \: \: 3) \\ \frac{3 - 1}{3 - ( - 1)} = \frac{2}{4} = \frac{1}{2} \)
The point (-sqrt2/2, sqrt2/2) is the point at which the terminal ray of angle theta intersects the unit circle. What are the values for cosine and cotangent functions for angle theta
Hence, the values of cοsine and cotangent functions for angle theta are -√(2)/2 and -1, respectively.
What precisely does a degree angle mean?Angles between 0 and 90 degrees are cοnsidered acute. Obtuse angles fall within this range and vary frοm 90° to 180°. A right angle is knοwn as an angle with a 90-degree angle (= 90°).
The pοint (-√(2)/2, √(2)/2) lies on the unit circle in the second quadrant, which means that the cοrresponding angle theta has a reference angle of pi/4.
The cοsine of theta is the x-coordinate of the point on the unit circle corresponding to theta. Since the x-coοrdinate of the given point is -√(2)/2, we have:
cοs(θ) = -√(2)/2
The cοtangent of theta is the ratio of the cosine of theta to the sine of theta. We can find the sine of theta by using the Pythagοrean theorem, since the pοint (-√(2)/2, √(2)/2) lies on the unit circle:
sin(θ) = √(1 - cos²(θ)) = √(1 - (-√(2)/2)²) = √(2)/2
Therefοre, we have:
cot(θ) = cos(θ) / sin(θ) = (-√(2)/2) / (√(2)/2) = -1
Hence, the values of cοsine and cotangent functions for angle theta are -√(2)/2 and -1, respectively.
To know mοre about angle visit:
brainly.com/question/28451077
#SPJ1
(square root)7 - x = x + 5. Find x
.We have the following equation,
\(\sqrt[]{7-x}=x+5\)First, we can note that the term into the radical must be zero or positive, that is,
\(\begin{gathered} 7-x\ge0 \\ \text{which means} \\ x\le7 \end{gathered}\)Then, by squaring both sides, we have
\(\begin{gathered} 7-x=(x+5)^2 \\ or\text{ equivalently,} \\ 7-x=x^2+10x+25 \end{gathered}\)Now, by subtracting 7 to both sides, we have
\(-x=x^2+10x+18\)and by adding x to both sides, we get
\(\begin{gathered} 0=x^2+11x+18 \\ or\text{ equivalently,} \\ x^2+11x+18=0 \end{gathered}\)Now, we can use the quadratic formula, that is,
\(x=\frac{-11\pm\sqrt[]{11^2-4(1)(18)}}{2}\)which gives
\(\begin{gathered} x=\frac{-11\pm\sqrt[]{121-72}}{2} \\ x=\frac{-11\pm\sqrt[]{49}}{2} \\ x=\frac{-11\pm7}{2} \end{gathered}\)then, the solutions are
\(\begin{gathered} x=\frac{-4}{2}=-2 \\ \text{and} \\ x=\frac{-18}{2}=-9 \end{gathered}\)Now, we know that an extraneous solution is a solution that does not work. Then, by substituting x=-9 into the given equation, we have
\(\sqrt[]{7-(-9)}=-9+5\)which gives
\(\begin{gathered} \sqrt[]{7+9}=-4 \\ \sqrt[]{16}=-4 \\ 4=-4 \end{gathered}\)which is an absurd result. Then, x=-9 is an extraneous solution.
On the other hand, by substituting x=-2, we obtain
\(\begin{gathered} \sqrt[]{7-(-2)}=-2+5 \\ \sqrt[]{7+2}=3 \\ \sqrt[]{9}=3 \\ 3=3 \end{gathered}\)which is correct.
Therefore, the answers are:
\(\begin{gathered} \text{Extraneous solution: x=-9} \\ \text{True solution: x=-2} \end{gathered}\)0.75 x - 4/3 Brainless?
Answer:
-1
Step-by-step explanation:
Turned 0.75 into fraction which is 3/4.
Then did 3/4x-4/3=-1
Evalúe the limit. Show steps
The value of the limit will be equal to -2.
A function may be defined as the one in which for one input variable x there is only one output variable y. The input variable is called independent variable and the output variable is called dependent variable. Limit on a function may be defined as the value of input variable reaches a certain specified value the output of function also reaches a certain specified value. The function \(\lim_{x \to-2}\) (x²)/(2x + 4) can be solved by using the rule \(\lim_{x \to a}\) f(x)/g(x) = \(\lim_{x \to a}\) f'(x)/g'(x).
Here f(x) = x², f'(x) = 2x
g(x) = 2x + 4, g'(x) = 2
Now, \(\lim_{x \to-2}\) (x²)/(2x + 4) = \(\lim_{x \to-2}\) (2x/2)
\(\lim_{x \to-2}\) (x) = -2 which is the required value.
Learn more about Limit at:
brainly.com/question/12017456
#SPJ1
according to a recent pew research center report, many american adults have made money by selling something online. in a random sample of 4579 american adults, 914 reported that they earned money by selling something online in the previous year. assume the conditions for inference are met. construct a 98% confidence interval for the proportion of all american adults who would report having earned money by selling something online in the previous year.
As per the 98% confidence interval for the proportion of all American adults who would report having earned money by selling something online in the previous year is (0.1859,0.2133).
Confidence interval:
In statistics, confidence interval is an estimate of an interval in statistics that may contain a population parameter.
Given,
According to a recent pew research center report, many American adults have made money by selling something online. in a random sample of 4579 American adults, 914 reported that they earned money by selling something online in the previous year. assume the conditions for inference are met.
Here we need to construct a 98% confidence interval for the proportion of all American adults who would report having earned money by selling something online in the previous year.
As per the given question, the value of
Confidence interval = 98%
Number of samples = 4579
Reported numbers = 914
Therefore, the number of failure is calculated as,
=> 4579 - 914
=> 3665
Here the sample proportion is the number of successes divided by the sample size:
=> 941/4579
=> 0.1996
Here for confidence level 1 − α = 0.98,
We have to determine zα / 2 = z0.01 using the normal probability table in the appendix, we get
zα/2 = 2.33
Then the margin of error is calculated as,
E = 2.33 x √[0.1996 x (1 - 0.1996)]/4579
E = 0.0137
Then the boundaries of the confidence interval are then:
p⁻ˣ = 0.1996−0.0137 = 0.1859
p⁺ˣ = 0.1996+0.0137 = 0.2133
To know more about confidence interval here.
https://brainly.com/question/24131141
#SPJ4
825 use each digit once. make the smallest 3digit number
Step-by-step explanation:
Given: To make smallest 3-digit number of 825.
To find: The smallest 3-digit number of 825.
Solution: We can make the smallest 3-digit number of 825 by separating the numbers and arranging it to ascending order. The given number is 825. ...
Final answer: The smallest 3-digit number of 825 is 258.
hope it helps
Answer:
258
Step-by-step explanation:
We are given 3 numbers:
8 2 5
And we are asked to find the smallest 3 digit number using those 3 digits above.
To make the smallest number, place the numbers in value from least to greatest:
2 5 8
This is your 3 digit number: 258.
Hope this helps! :)
what is the area measured and square centimeters of square picture below
Y’all keep on getting it wrong so do it right
Answer:
But it's your work so you shouldn't be the one complaining. You are depending on us.
Btw, the answer is deleted.
Answer:
the answer is lemme cop these 5 points real quick
The pattern 96, 92, 88, 84, ________ follows the "subtract 4" rule. Study the pattern to find traits that are not obvious in the rule. Explain.
pleas in one or 2 sentences
PLEASE WHAT IS THIS?!!? WHATS THE ANSWER?
Answer: bottom right
which number line shows the solution to x + 8 > 15
7 is not included and hence it is depicted on the number line using an open dot. Hence, solution to the inequality is option B.
What is inequality?In mathematics, inequalities specify the connection between two non-equal numbers. Equal does not imply inequality. Typically, we use the "not equal sign (≠)" to indicate that two values are not equal. But several inequalities are utilized to compare the numbers, whether it is less than or higher than.
The given inequality is:
x + 8 > 15
Subtracting 8 on both sides of the equation:
x + 8 - 8 > 15 - 8
x > 7
Here, 7 is not included and hence it is depicted using an open dot.
Hence, option B is the correct answer.
Learn more about inequality here:
https://brainly.com/question/28823603
#SPJ1
A local pizzeria offers 14 toppings for their pizzas and you can choose any 4 of them for one fixed price. How many different types of
pizzas can you order with 4 toppings?
0 24,024
3,632,428,800
1,001
0 1
Answer:
Hi rei its cam
Step-by-step explanation:
Reniel is 13 years old. He is two years younger than his brother, Gelo. If Gelo is five times older than their sister, how old is their sister?
How much simple interest is earned on $4,000 in 3 1/2 years at an interest rate of 5.2%?
Answer:
$728
Step-by-step explanation:
The formula for simple interest is I = PRT, where I = interest earned/paid, P = principal amount deposited or borrowed, R = rate of interest as a decimal, and T = time in years.
I = PRT
I = (4000)(0.052)(3.5)
I = 728
Simple Interest earned on $4000 in three and half year with 5.2% interest rate is $ 728.
What is Simple Interest?Simple interest is a method to calculate the amount of interest charged on a sum at a given rate and for a given period of time.
Here, Principal = $ 4000
Time = \(3\frac{1}{2}\) years = 7/2 years = 3.5 years
Rate = 5.2 % = 0.052
Now, SI = P.R.T/100
SI = 4000 X 3.5 X 5.2 / 100
SI = $ 728
Thus, Simple Interest earned on $4000 in three and half year with 5.2% interest rate is $ 728.
Learn more about Simple Interest from:
https://brainly.com/question/22621039
#SPJ2
Which point is closest to the y-axis?
(10, 15)
(5, 12)
(9, 11)
(-4, 14)
Consider the function below, which has a relative minimum located at (-3, -18) and a relative maximum located at 1/3, 14/27). f(x) = -x3 - 4x2 + 3x. Select all ordered pairs in the table which are located where the graph of f(x) is decreasing: Ordered pairs: (-1, -6), (2, -18), (0, 0),(1 , -2), (-3 , -18), (-4. , -12)
The ordered pairs (-1, -6), (2, -18), (0, 0), and (-4, -12) do not correspond to the intervals where the graph of f(x) is decreasing. The pairs (1, -2) and (-3, -18) are the correct ones.
To determine where the graph of f(x) is decreasing, we need to examine the intervals where the function's derivative is negative. The derivative of f(x) is given by f'(x) = -3x^2 - 8x + 3.
Now, let's evaluate f'(x) for each of the given x-values:
f'(-1) = -3(-1)^2 - 8(-1) + 3 = -3 + 8 + 3 = 8
f'(2) = -3(2)^2 - 8(2) + 3 = -12 - 16 + 3 = -25
f'(0) = -3(0)^2 - 8(0) + 3 = 3
f'(1) = -3(1)^2 - 8(1) + 3 = -3 - 8 + 3 = -8
f'(-3) = -3(-3)^2 - 8(-3) + 3 = -27 + 24 + 3 = 0
f'(-4) = -3(-4)^2 - 8(-4) + 3 = -48 + 32 + 3 = -13
From the values above, we can determine the intervals where f(x) is decreasing:
f(x) is decreasing for x in the interval (-∞, -3).
f(x) is decreasing for x in the interval (1, 2).
Now let's check the ordered pairs in the table:
(-1, -6): Not in a decreasing interval.
(2, -18): Not in a decreasing interval.
(0, 0): Not in a decreasing interval.
(1, -2): In a decreasing interval.
(-3, -18): In a decreasing interval.
(-4, -12): Not in a decreasing interval.
Therefore, the ordered pairs (-1, -6), (2, -18), (0, 0), and (-4, -12) are not located in the intervals where the graph of f(x) is decreasing. The correct answer is: (1, -2), (-3, -18).
For more question on intervals visit:
https://brainly.com/question/30460486
#SPJ8
Note the complete and the correct question is
Q- Consider the function below, which has a relative minimum located at (-3, -18) and a relative maximum located at 1/3, 14/27).
\(f(x) = -x^3 - 4x^2 + 3x\).
Select all ordered pairs in the table which are located where the graph of f(x) is decreasing: Ordered pairs: (-1, -6), (2, -18), (0, 0),(1 , -2), (-3 , -18), (-4. , -12)
The beginning balance of the check register was $492.88. During the month, deposits were made for $210 and $499.88. Checks were
written for $18.25, $19.82, and $309.82. What is the ending balance of the check register?
Note: Round your answer to 2 decimal places.
Answer:
$840.77
Step-by-step explanation:
9x+10y =1 9x-6y=-15 solve using elimination
Answer:
I. x = -1
II. y = 1
Step-by-step explanation:
Given the following algebraic expressions;
9x + 10y = 1 .......equation 1
9x - 6y = -15 ......equation 2
To solve using the elimination method, we would subtract eqn 1 from eqn 2.
(9x - 9x) + (-6y - 10y) = (-15 - 1)
0x - 16y = -16
-16y = -16
16y = 16
y = 16/16
y = 1
Next, we would find the value of x;
From eqn 1;
9x + 10y = 1
Substituting the value of y, we have;
9x + 10(1) = 1
9x + 10 = 1
9x = 1 - 10
9x = -9
x = -9/9
x = -1
CAN SUM1 please help mee I need to turn it in in 5min!♥️
Answer:
x = 36
1 = 42
2 = 108
Step-by-step explanation:
2x + x + 72 = 180
3x + 72 = 180
-72 -72
3x = 108
divide by 3
x = 36
_______________________
1 = 2x
1 = 2(36)
1 = 42
_______________________
2 = x + 72
2 = 36 + 72
2 = 108
Hope this helped! Have a nice day! Plz mark as brainliest!!!
-XxDeathshotxX
You are writing music for a movie and you have to synchronize the music to the amount of frames per click in it. It's a battle scene so you want fast, energetic and exciting music. You choose a Presto tempo marking of 200 beats per minute. How many picture frames are there per each tempo click? (Round to the nearest whole number and write only the number.)
Answer: 7.2 frames per bit.
Step-by-step explanation:
Our teempo is 200 bpm.
in one minute we have 60 seconds, so here we have:
200b/60s = 3.33 bits per second.
For movies, the standar is 24 frames per second
now, we can take the quotient between the frames per second and the bits per second and get the frames per bit.
24fps/3.33bps = 7.2 frames per bit.
A number line going from negative 3 to positive 5. a closed circle is at negative 1.5. everything to the left of the circle is shaded. which inequalities have the solution set graphed on the number line? check all that apply. x greater-than-or-equal-to -1.5 x less-than-or-equal-to -1.5 -1.5 less-than-or-equal-to x -1.5 greater-than-or-equal-to x x less-than-or-equal-to -2.5 x greater-than-or-equal-to -2.5 -2.5 greater-than-or-equal-to x
The inequalities which matches the graph are: x ≥ ₋1.5 and ₋1.5 ≤ x
Given, a number line is moving from ₋3 to ₊5 .
Next a mark is made at ₋1.5 and everything to its left is shaded which means not visible.
When we mark the point and shade the left part of it then we can start applying the inequality expressions.
And from that we can match the applicable inequalities while observing the graph.
For the first inequality ₋1.5 ≥ x.Here,x value ranges from ₋1.5 to ₊5, hence we take this as an inequality expression.Next, if we consider x ≤ ₋1.5, then here x value will range from ₋1.5 to ₋3. where the region is shaded. Hence this expression doesn't satisfy the graph.the next expression is ₋1.5 ≤ x. here the value will again range in the shaded area so it is not applicable.₋1.5 ≥ x, here the values will satisfy the graph.remaining inequality expressions does not support the graph.Therefore the only inequalities the graph represents is x ≥ ₋1.5 and ₋1.5 ≤ x
Learn more about "Linear Inequalities" here-
brainly.com/question/371134
#SPJ10
Solve for x: 5x + one third(3x + 6) > 14 x > twelve fifths x > 2 x < twelve fifths x < 2
The solution of x in the inequality is x > 2
How to solve for x in the inequality?The inequality is given as:
5x + one third(3x + 6) > 14
Rewrite properly as:
5x + 1/3(3x + 6) > 14
Open the brackets
5x + x + 2 > 14
Subtract 2 from both sides of the inequality
5x + x + 2 - 2> 14 - 2
Evaluate the difference
5x + x > 12
Evaluate the like terms
6x > 12
Divide both sides of the inequality by 2
x > 2
Hence, the solution of x in the inequality is x > 2
Read more about inequality at:
https://brainly.com/question/24372553
#SPJ1
Answer:
x > 2, like the other guy said
Step-by-step explanation:
I hope this helps
The risk of a child developing cancer is approx 3 in 1500. If there are approx 11,721,722 children, how many have cancer?
Answer:
Approximately 23,433 children will have cancer.
Step-by-step explanation:
3/1500 can be simplified to 1/500, which can also be written as 0.002. To find the number of children who have cancer, we do 11,721,722 * 0.002, which gives us 23,433.444 which we can round to 23,433.
what does 5 × 100 equal
Answer: 500.
Step-by-step explanation:
Your answer is 500.